Step 2 CK (CCSSA) Form 6

These answer explanations are and always will be free. However, given multiple email requests, I will post my Venmo (@Adam-Zakaria-SLO) if you want to send a few dollars to show your support for the website.

I also offer reasonably priced Study Guides and Personalized Study schedules, so please reach out using the Tutoring menu option or Study Guides and Personalized Study schedules menu option listed above if you would like personalized support.

Furthermore, I offer personal statement and application review services for residency applicants, so please reach out using the “Residency Advising and Application Preparation” menu option above if interested.

Lastly, please check out my Youtube channel (https://www.youtube.com/channel/UCT1Ukl4pm5QK9iw6h4MB_Hw/playlists) and the “Biostatistics Curriculum” option above for free videos and practice questions reviewing all the essential biostatistics topics covered on NBME exams. Good luck with your exams!

Exam Section 1:

1. A 14-month-old girl is brought to the physician because of a 14-hour history…

Water-soluble contrast enema

  • Intussusception often presents in a child 6-36 months old with abdominal pain relieved by drawing knees to the chest +/- bloody diarrhea +/- vomiting. In setting of suspected intussusception, next best step is a contrast/air enema which is both therapeutic and diagnostic.
  • X-ray of upper GI tract with contrast indicated in setting of suspected malrotation with volvulus (newborn with abdominal pain + bilious vomiting + distention)
  • Corticosteroid enemas sometimes used in ulcerative colitis

2. A 52-year old woman comes to the physician because…

Decreased androgens

  • Ovaries not only produce estrogens, but also produce androgens that play a central role in libido (among other things)

3. A 27-year old man comes to the physician because of a 1-week history…

Diffuse hypokinesia and dilation of the ventricles

  • Young patient who presents with acute signs of heart failure (elevated JVP, bilateral basilar crackles, displaced PMI, S3, lower extremity edema, etc.) most likely represents myocarditis, especially in setting of recent viral URI
  • Asymmetric septal hypertrophy = HOCM = Sudden cardiac death or exertional syncope in young adult
  • Bicuspid aortic valve with stenosis presents as symptomatic AS (syncope, angina, dyspnea) often in a 50-60 year old

4. During the past 4 months, a 32-year-old woman has had pain…

Sjogren syndrome

  • Mucosal dryness in young or middle-aged woman, especially in one with other autoimmune disease such as this patient (T1DM), should make you think of Sjogren syndrome
  • Important to contrast with age-related sicca syndrome, which is present in ~25% of individuals over 65 years old and leads to sensation of dry eyes and/or dry mouth with a negative ANA (if they are feeling generous and give you this information)

5. A 24-year-old primigravid woman at 18 weeks’ gestation…

Inflammatory bowel disease

  • IBD should always be suspected in a young adult without a clear trigger (international travel, etc.) who presents with increased bowel movements, especially if stools are covered with blood and/or patient has associated abdominal pain
  • Diseases associated with erythema nodosum = IBD, tuberculosis, systemic fungal infections (cocci, etc.), sarcoidosis, Behcet’s disease, strep pharyngitis (most common trigger)
  • Important to distinguish IBD from IBS because both can present in young adults and both can be described as having increased frequency of mucus-covered stools. Features that point towards IBS include lack of blood in stools, abdominal pain that is relieved with defecation and alternating periods of constipation and diarrhea

6. A 67-year-old man comes to the emergency department 1 hour…

Right vertebral

  • Crossed signs of decreased pinprick sensation over opposite sides of face and body = Pathology in the brainstem IPSILATERAL to side where face pinprick sensation is lost
  • Other aspects of patient’s presentation are consistent with Wallenberg syndrome (stroke of lateral medulla), including vertigo, ataxia and weakness of the palate
  • Key idea: If patient with suspected brainstem stroke has symptoms that localize to nucleus ambiguus (motor vagal nucleus that when damaged can present as dysphagia, hoarseness, and dysarthria) then you are almost certainly dealing with a lateral medullary syndrome
  • Although lateral medullary syndrome most often associated with stroke/hemorrhage involving the Posterior Inferior Cerebellar artery (PICA), important to remember that the PICA comes off the vertebral artery (reason why vertebral dissections can present with lateral medullary syndrome)

7. A 25-year-old woman comes to the physician because…

Complete blood count

  • Appetite for non-nutritive substances (such as ice, hair, cornstarch on its own) = pica
  • Most commonly associated with iron deficiency anemia, with other clues in the stem being the mild pallor on physical exam and leiomyomata uteri (which often lead to heavy menstrual bleeding)

8. A 28-year-old woman at 28 weeks’ gestation…

Mitral stenosis

  • Diastolic murmur = Aortic regurgitation or mitral stenosis (assuming left-side of heart)
  • AR murmur = Early, diastolic, decrescendo murmur heard best at upper sternal border
  • MS murmur = Diastolic snap (“sharp sound after S2”) followed by a late diastolic murmur heard best at the apex that can sometimes be described as increasing in intensity before S1 (due to atrial kick pushing increased blood across valve right before valve closes)
  • Key idea: In setting of pregnant woman with new-onset dyspnea, always consider mitral stenosis (increased cardiac output due to plasma expansion in pregnancy can cause previously asymptomatic valvular defect to become symptomatic) and pulmonary embolism (pregnancy is a hypercoagulable state)

9. A 37-year-old woman is brought to the emergency department…

Right optic nerve

  • Young woman + eye pain with ipsilateral vision problems = Optic neuritis
  • Optic neuritis (as the name implies) is due to inflammation of the associated optic nerve, with further proof that optic nerve is involved being the relative afferent pupillary defect (problem with light getting back to retina in this case)

10. A 70-year-old woman has had increasing abdominal pain…

Gram stain of abdominal fluid

  • Patient with ascites who presents with diffuse abdominal pain/tenderness + fever + leukocytosis = Spontaneous bacterial peritonitis
  • Diagnosis of SBP is actually typically made after paracentesis if the ascitic fluid has >250 PMNs (neutrophils) per mL
  • Although we often think of Spontaneous Bacterial Peritonitis (SBP) as being a disease exclusively seen in patients with cirrhosis, in reality it can affect patients with any conditions that lead to ascites (fluid in the abdomen), with patients who undergo peritoneal dialysis being at higher risk given the fact that significant opportunities for contamination during catheter exchanges, etc.

11. A 32-year-old woman comes to the physician because of…

Thyroid-stimulating hormone

  • Difficulty of this question is trying to differentiate between normal stresses of being a new mother vs. hypothyroidism
  • Features that point to hypothyroidism vs. normal stress include elevated cholesterol levels, memory problems, and lethargy
  • Postpartum thyroiditis can present up to 1 year after giving birth and although it is a fairly rare disease in real life, it is often tested on these exams
  • Cortisol = Cushing’s (gaining weight, hirsutism, etc.) or Adrenal insufficiency (hypotensive, hyperpigmented skin, craving salt, etc.)

12. A 67-year-old woman is hospitalized because of abdominal pain …

Intramural hematoma of the proximal small bowel

  • Patient recently started on anticoagulation with a severely elevated INR (therapeutic range is 2-3) and rapidly dropping hemoglobin

13. A previously healthy 32-year-old woman comes to the physician …

Inhaled Beta-2 adrenergic agonist

  • 3 common causes of a nighttime cough in an otherwise healthy person include asthma, GERD and post-nasal drip
  • This patient’s presentation is most consistent with cough-variant asthma likely exacerbated by recent URI because of the end-expiratory wheezes on physical exam and decreased expiratory flow rate
  • GERD would more commonly lead to burning chest pain or sour taste, whereas post-nasal drip would lead to cobblestoning of the throat and would be associated with other allergy symptoms (runny nose, itchy eyes, etc.)

14. A 27-year-old primigravid woman at 38 weeks’ gestation ….

Recommend autopsy of the infant

  • Random question that could be approached through process of elimination (inconsiderate to tell mother about becoming pregnant again right after losing child, don’t know risk of recurrence without knowing cause, patient presented with baby already having lack of fetal heart tones so nothing with hospital care led to outcome, etc.)

15. A 25-year-old woman comes to the physician for a routine…

Borderline personality disorder

  • Patient has persistent history of unstable relationships and self-injurious behaviors
  • Delusional disorder and adjustment disorder are transient pathologies
  • Major depressive disorder = Depressed mood and SIGECAPS

16. A 15-year-old girl is brought to the physician by her parents…

Negotiate a contract regarding medication compliance

  • Another random question that can be approached through process-of-elimination
  • Patient doesn’t need any changes to medication regimen because her problem is compliance, cannot ethically refuse to see patient because of compliance, patient has no psychiatric issues, and unethical for parents to punish patient for lack of compliance

17. A previously healthy 52-year-old man comes to the physician…

Insulin

  • Clinical picture consistent with Type 2 diabetes (older overweight man with polyuria, nocturia and increased non-fasting serum glucose concentration)
  • Key idea: Early on in Type 2 diabetes, insulin levels are high because the pancreas is trying to make up for the insulin-resistance
  • Key idea: Later on in Type 2 diabetes (on the scale of years after initial diagnosis), insulin levels become low as the beta cells become burned out from overproducing insulin for so long and secondary to amylin deposition within pancreas (amyloidosis)

18. A healthy 18-year-old woman comes for a routine health…

Testing for Neisseria gonorrhoeae and Chlamydia trachomatis

19. A 5-year-old boy is brought to the emergency department 30 …

Hyaline casts

  • Hyaline casts are the most common type of urinary cast and are often seen in setting of dehydration or vigorous exercise; they can be seen in healthy individuals
  • Erythrocyte casts = RBC casts = Glomerulonephritis (such as post-strep glomerulonephritis)
  • Leukocyte casts = WBC casts = Pyelonephritis and acute interstitial nephritis

20. A 37-year-old woman comes to the physician because of progressive…

Pulmonary artery pressure

  • Classic presentation of mitral stenosis on NBME exams (rheumatic heart disease, worse with pregnancy, murmur description with diastolic snap-murmur)
  • Over time, mitral stenosis will lead to increased pressures in the left atrium which will back up into the lungs and the right side of the heart, eventually leading to right-sided heart failure; this knowledge coupled with the right ventricular lift (which occurs because the RV experiences hypertrophy secondary to increased pulmonary artery pressure) points to elevated pulmonary artery pressure
  • Diastolic filling time decreased (takes longer for mitral valve to open) and LV end-diastolic pressure also decreased/normal because of less blood flow from LA to LV

21. Two hours ago, a 24-year-old man had the sudden…

Tube thoracostomy

  • Tall, young men are prone to spontaneous pneumothorax, with the chest x-ray in this stem showing a right-sided pneumothorax (vascular lines don’t extend to periphery and you can actually see a clear outline of the collapsed lung within the lung field)
  • A small (<2 cm), stable, asymptomatic pneumothorax can often be managed conservatively with observation (https://www.nejm.org/doi/10.1056/NEJMoa1910775)
  • The patient in this stem is symptomatic and having difficulty breathing, so in this case we would want to either perform a tube/needle thoracostomy
  • -ostomy = making a hole, whereas -otomy = cutting

22. A 42-year-old man is brought to the emergency department …

Vitamin B1 (thiamine)

  • Wernicke encephalopathy is due to thiamine deficiency, often seen in alcoholics (which is a common NBME way of suggesting patient may have nutritional deficiency) and leads to triad of confusion, ataxia and ophthalmoplegia
  • Key idea: Patients presenting to ED with altered mental status of unknown etiology often given glucose, thiamine and naloxone, but a commonly tested concept on NBME exams is that patients should get thiamine before glucose because glucose first can induce Wernicke encephalopathy in patient with underlying nutritional deficiency
  • Chlordiazepoxide would be used in patient with alcohol withdrawal or delirium tremens (autonomic hyperactivity would be more prominent in presentation)

23. A previously healthy 10-year-old boy is brought to the physician…

Avoidance of the wooded area

  • Patient most likely has poison oak/ivy, which presents >1 day after exposure (Type 4 hypersensitivity) and will lead to a sharp line between rash and unaffected skin
  • Patients do not receive prophylaxis against poison oak/ivy
  • Cat scratch can lead to cat scratch fever, which leads to lymphadenopathy but will not lead to vesicles/bullae

24. An otherwise healthy 4-month-old girl is brought to the physician…

No intervention is necessary

  • Presentation most consistent with a hemangioma of the skin, which commonly grows during the first year of life but involutes on its own within 5 years
https://clinicalgate.com/infantile-hemangiomas-and-vascular-malformations/

25. A 3-year-old boy is brought to the physician because of a 7-day…

Impaired phagocyte oxidative metabolism

  • Presentation most consistent with chronic granulomatous disease which is most commonly X-linked (maternal uncle affected) and leads to recurrent skin abscesses with aspiration showing neutrophils filled with bacteria (able to take up bacteria, but can’t perform oxidative burst to kill bacteria)
  • X-linked immunodeficiencies: Bruton agammaglobulinemia, Severe combined immunodeficiency, Hyper-IgM syndrome, Wiskott-Aldrich syndrome, Chronic granulomatous disease

26. A 2-month-old boy is brought to the physician:

Language development: Normal // Psychosocial development: Normal

  • Language development: Babies “coo” around 2 months but don’t laugh until 3-4 months
  • Psychosocial development: Babies smile around 2 months, but won’t reach for toys until 3-4 months

27. A 72-year-old man comes to the emergency department after…

Corticosteroid therapy

  • Presentation most consistent with temporal arteritis (older patient with headache, jaw claudication, elevated ESR and tender, enlarge temporal arteries)
  • Key idea: Because of risk for blindness and imperfect sensitivity of temporal artery biopsy (because disease only affects section of temporal artery so biopsy may miss affected segment), patients with suspected temporal arteritis should begin empiric treatment with corticosteroids BEFORE temporal artery biopsy is performed

28. Four hours after undergoing a cesarean delivery at term…

Postoperative intra-abdominal hemorrhage

  • Patient’s blood pressure has dropped, pulse has gone up and hematocrit has dropped relative to intraoperative values, pointing to bleeding after surgery
https://www.quora.com/How-much-blood-do-you-have-to-lose-to-start-feeling-dizzy-and-cause-a-hypovolemic-shock

29. A 32-year-old woman comes to the physician because of a 1-year…

Histocompatibility human leukocyte antigen B27

  • Presentation most consistent with ankylosing spondylitis (young person with inflammatory back pain (stiff for >1 hour upon waking and improves with movement) who specifically has involvement of sacroiliac joints)
  • HLA-B27 positivity is a risk factor for PAIR diagnoses (Psoriatic arthritis, Ankylosing spondylitis, IBD related inflammatory arthritis and Reiter/Reactive arthritis)
  • Patient would have elevated ESR

30. A 32-year-old woman comes to the physician because she…

Polycystic ovarian syndrome

  • One of the most common causes of female infertility, especially in overweight women
  • Diagnosis requires 2 out of 3 of the following criteria: (1) Clinical and/or biochemical hyperandrogenism (2) Oligomenorrhea (3) Polycystic ovaries on pelvic ultrasound /// Our patient has clinical hyperandrogenism (acne vulgaris) and oligomenorrhea (irregular menses)
  • Key idea: Although often associated with insulin resistance and ultimately associated with development of Type 2 diabetes, insulin resistance is not formally included in diagnostic criteria

31. A 57-year-old man is brought to the emergency department 30…

Intravenous calcium gluconate

  • Patient has missed several days of dialysis and was found down in his house (which can lead to rhabdomyolysis), giving this patient 2 reasons to have metabolic acidosis with hyperkalemia
  • Hyperkalemia can lead to abnormal heart rhythms that can lead to patient death, so patients who have hyperkalemia with one of the following three features should first receive calcium gluconate to stabilize the cardiac membrane prior to trying to reduce potassium levels: (1) Potassium > 7.0 (2) ECG changes (Peaked T waves, PR prolongation and QRS widening, disappearance of P wave) (3) Rapidly rising potassium due to tissue breakdown or tumor lysis syndrome

32. A 32-year-old man has had mild tremulousness, increasing…

Discontinuation of lorazepam

  • Alcohol and benzodiazepine withdrawal lead to similar clinical syndromes with increased autonomic activity (tremors, anxiety, sweating, agitation, etc.) because prolonged use of these medications leads to downregulation of GABA receptors, such that removal of GABA agonists (alcohol, benzo’s) leads to relatively decreased inhibitory activity from ligand binding to GABA receptors
  • Key idea: Withdrawal often leads to the opposite effects of intoxication with drug (Example: Opiate use leads to constipation and pinpoint pupils, whereas opiate withdrawal leads to diarrhea and mydriasis)
  • https://www.mdcalc.com/ciwa-ar-alcohol-withdrawal

33. A 20-year-old man comes for a routine health maintenance…

Myotonic muscular dystrophy

  • Autosomal dominant trinucleotide repeat disease (father also affected) that classically leads to myotonia (difficulty relaxing muscles, most classically gripping) + CTG (Cataracts, Toupee (early balding) and Gonadal atrophy
  • Key idea: These patients also commonly have psychosocial symptoms (irritability, tired during day, etc.)

34. An 18-month-old boy is brought to the emergency department…

Passive hypersupination of the forearm

  • Presentation most consistent with radial head subluxation, which is where pulling on a young child’s arm causes the radial head to be trapped within the annular ligament
  • Patient’s will classically be holding their arm in an extended and pronated position with no visible deformity of elbow
  • Can be treated by either (1) Hyperpronation of forearm OR (2) Supination of forearm and flexion of elbow

35. A 4-year-old boy is brought to the emergency department…

Pulmonary contusion

  • Patient with blunt thoracic trauma who presents <24 hours later with symptoms (tachypnea, tachycardia, hypoxia) and/or CT/CXR infiltrates over affected area (patchy, alveolar)
  • Key idea: Important to differentiate from ARDS, which can have similar presentation but often occurs 24-48 hours after trauma

36. An asymptomatic 57-year-old man comes to the physician…

Accumulation of lipids in the arterial wall

  • Abdominal bruit + hypertension = Renal artery stenosis
  • Most common causes of renal artery stenosis include atherosclerosis, fibromuscular dysplasia and polyarteritis nodosa
  • Our patient has clear risk factors (middle-aged man with significant smoking history and likely other poor health behaviors) for atherosclerosis, which is the most common cause overall
  • Fibromuscular dysplasia often seen in otherwise healthy young woman presenting with new hypertension or signs of poor brain perfusion
  • Polyarteritis nodosa often seen with abdominal pain in a middle-aged man with hepatitis B (or risk factors for Hep B)

37. A 20-month-old girl is brought to the physician because of …

Cystic fibrosis

  • Patient of Northern European descent with recurrent sinopulmonary infections, poor growth (due to fat malabsorption) and signs of bronchiectasis (streaky lung densities with hyperinflation, mild clubbing)
  • Important to distinguish from Kartagener’s syndrome, which would lead to dextrocardia and would be less likely to lead to poor weight gain

38. A previously healthy 16-year-old boy is brought to the physician…

Pulmonary tuberculosis

  • Patient with clear exposure (Albania and other Eastern European countries are endemic TB countries) showing signs of infection (cough and fever) with adenopathy
  • Pulmonary aspergillosis would more commonly be seen in somebody who is immunocompromised (acute angioinvasive aspergillosis) or somebody with cystic fibrosis or asthma (allergic bronchopulmonary aspergillosis)

39. A 24-year-old man is brought to the emergency department...

Observation in the emergency department

  • Presentation most consistent with marijuana intoxication (injected conjunctiva, inappropriate laughter, increased appetite, paranoia) which is self-limited and managed with observation

41. An 82-year-old woman is brought to the physician by her granddaughter…

Major depressive disorder

  • Differential for dementia in elderly patient includes: Alzheimer’s (chronic neurodegenerative course), Vascular dementia (patient with ASCVD risk factors with stepwise decline), Frontotemporal dementia (early personality changes often around 60 yo), Dementia with Lewy bodies (parkinsonism, visual hallucinations, fluctuating cognition), Normal pressure hydrocephalus (“wet, wacky and wobbly”) and pseudodementias (depression, hypothyroidism, B12 deficiency)
  • In this patient with acute/subacute forgetfulness along with components of SIGECAPS (trouble sleeping, decreased appetite, psychomotor retardation) and history of past episodes of forgetfulness that were able to be treated with medication most consistent with a major depressive disorder episode
  • Key idea: Alzheimer’s can also lead to signs/symptoms similar to Major depressive disorder, but Alzheimer’s would have a chronic progressive course rather than an episodic course as is seen in this question stem

42. A 42-year old computer science professor is brought to the…

Huntington’s disease

  • Patient between 40-60 years old often with family history of disorder or unknown family history who presents with triad of chorea, depression and dementia
  • Key idea: Patients can also commonly have psychosis

43. A 77-year-old woman comes to the physician because of a 2-day…

Gallstone ileus

  • Patient has signs of a small bowel obstruction (abdominal distention, abdominal pain, vomiting, high-pitched bowel sounds with air-fluid levels on x-ray)
  • Most common causes of small bowel obstruction are adhesions, hernia and malignancy, but this patient has air in the liver which is basically pathognomonic for gallstone ileus (can also have air in the biliary tree)
References: Netter’s Atlas of Human Anatomy

44. A 67-year-old man is examined 3 days after undergoing…

Remove the urinary catheter

  • Most common types of hospital-acquired infections are central-line associated bloodstream infections (CLABSI), Catheter-associated Urinary tract infections (CAUTI), Surgical Site Infection and Ventilator-associated pneumonia
  • Best way to avoid Catheter-associated UTI is by removing the catheter as soon as possible, which is appropriate in this patient because they have normal mental status and are able to move
  • Key idea: Prophylactic antibiotics and bladder irrigation with antibacterial solutions can increase the risk of developing resistant organisms without offering much benefit for reducing infections

45. A 6-month-old girl is brought to the physician because of poor…

Patent ductus arteriosus

  • Leads to a widened pulse pressure (because blood flows from aorta to pulmonary arteries during diastole, leading to reduced systemic diastolic pressure) with a continuous to-and-fro murmur
  • Key idea: PDAs may also predispose to an increased risk of respiratory/lung infections

46. A program for the primary prevention of coronary artery…

Incidence

  • Incidence > Prevalence because you are concerned about development of new cases (primary prevention)
  • Incidence = Number of new cases over a time period
  • Prevalence = Number of cases in the population
  • Mortality, case fatality and hospitalization would be more reflective of tertiary disease prevention efforts

Exam section 2:

1. A 22-year-old woman comes to the physician for a health…

Administration of hepatitis B vaccine

  • Patients who are adequately vaccinated against hepatitis B vaccine will have negative HBsAg, positive HGsAb and negative Hepatitis B core antibody (positive Hep B core antibody = past or current infection)
  • Patient is adequately protected against Hepatitis A due to presence of IgG antibodies, which can be the result of vaccination or past infection
  • Key idea: Indications for Hep A vaccination are chronic liver disease or increased risk of contracting disease (men who have sex with men, IVDU, travel to country where hepatitis A is prevalent, etc.)

2. A 5-month-old boy is brought for a follow-up examination. He…

Compression of the airway by a vascular ring

  • Patient has persistent wheezing despite treatment with bronchodilators, meaning that asthma is unlikely
  • 3 causes of chronic stridor/wheezing in a newborn/infant include laryngomalacia (inspiratory stridor/wheezing that worsens when feeding, crying or supine and improves when prone), vascular ring (biphasic stridor/wheezing that improves with neck extension) and airway hemangioma (worsening biphasic stridor over first year of life in patient with concurrent skin hemangioma)
  • Patients with proetin-induced allergic proctocolitis secondary to milk consumption often have bloody stools in setting of other allergic conditions (allergies, eczema or asthma)

3. A 37-year-old man is brought to the emergency department 6

Laparotomy

  • Key idea: Peritonitis (rigidity, guarding) + Free air under diaphragm = Emergency laparotomy!!!!!

4. An asymptomatic 32-year-old man comes for a routine health…

Selective IgA deficiency

  • Selective IgA deficiency commonly leads to sinopulmonary and GI infections (especially Giardia) and pathognomonically anaphylaxis to blood products (because patient develops anti-IgA antibodies and reacts against IgA in blood products)
  • Key idea: Most common immunodeficiency and is most commonly asymptomatic

5. Five weeks after vaginal delivery of a healthy full-tern newborn…

Admission to the hospital for treatment

  • Peripartum mood disorders are commonly seen within 4 weeks delivery
  • Most common is Postpartum blues (50-75% of pateints) who will have depressed affect and fatigue, but will have resolution within 2 weeks delivery
  • The most severe (and least common) peripartum mood disorder is postpartum psychosis, which often leads to delusions, hallucinations and thoughts of suicide or infanticide; these patients require immediate hospitalization due to high risk of injury to self or baby

6. A 17-year-old boy comes to the physician because of a 2-day history…

Inadequate production of insulin

  • Classic presentation of diabetic ketoacidosis in the setting of Type 1 diabetes (polyuria, polydipsia, tachypnea (to compensate for metabolic acidosis), fatigue/weakness, elevated glucose (often 300-500 mg/dL) and elevated ketones)
  • Younger patients can also develop type 2 diabetes, but that would more commonly be seen in a patient with elevated BMI and Type 2 diabetes almost never presents as DKA

7. A 62-year-old woman is admitted to the hospital because of…

Glucocorticoid-induced demargination and storage pool release of segmented neutrophils

  • Patient presented with a COPD exacerbation without overt infection (normal WBC with normal neutrophil count, afebrile) and then after receiving IV methylprednisolone (strong systemic steroid) developed a neutrophilic leukocytosis despite clinical improvements
  • Key idea: Two common causes of a neutrophilic leukocytosis are infection and systemic steroid use

8. A previously healthy 4-year-old boy is brought to the physician…

  • Unable to access media for this question, but patient likely had a no murmurs or a soft systolic ejection murmur in the setting of a clinical URI
  • CHF = S4 heart sound and crackles/rales (patient would have signs of edema)
  • Idiopathic pulmonary hypertension: Loud S2 and RV heave (patient would have chronic progressive dyspnea)
  • Pericarditis: Three-component friction rub (history would involve positional chest pain)
  • Ventricular septal defect: Common heart abnormality that would lead to a systolic murmur over the left lower sternal border, but would not explain the patients fever, cough and runny nose

9. A 3-month-old boy is brought for a well-child examination…

Glycogen storage disease, type 2 (Pompe’s disease)

  • Classically leads to cardiomegaly and hypotonia (including poor head control) in a young child
  • Muscular dystrophy would often present later on (toddler) and wouldn’t lead to heart symptoms early on in disease course
  • GM1 gangliosidosis [similar to Tay-Sachs (progressive neurodegeneration)] and infant botulism wouldn’t lead to heart symptoms

10. A previously healthy 4-year-old girl is brought to the physician…

Arthrocentesis

  • In children with new development of joint pain, two major diagnoses to consider are septic arthritis and transient synovitis
  • Our patient’s presentation more consistent with septic arthritis due to fever and refusal to bear weight, and therefore patient requires immediate drainage of joint space even before administering antibiotics because of risk for persistent damage to joint
  • Transient synovitis: Afebrile, history of previous viral URI, normal ESR, normal WBC, normal joint aspiration, self-limited condition

12. A 24-year-old woman comes to the physician because of constant, severe…

Fibromyalgia

  • Fibromyalgia classically leads to chronic widespread pain + tenderness at trigger points (pathognomonic) + normal labs and is often seen in anxious young patients (which is suggested by this patient’s diagnosis of irritable bowel syndrome)
  • Polymyositis: Proximal muscle weakness + elevated muscle enzymes + other signs of inflammation (ESR, CRP, etc.)
  • Polymyalgia rheumatica: Older patients with stiffness > pain and elevated markers of inflammation (ESR, CRP); important association with giant cell arteritis

13. A 67-year-old man is brought to the emergency department 4…

Ruptured aortic aneurysm

  • Older patient with sudden onset back pain unrelated to activity with signs of blood loss (pale, tachycardic, diaphoretic) who also has calcifications anterior to vertebral bodies on X-ray (aorta sits just anterior to vertebral bodies)

14. A 25-year-old woman comes to the physician for evaluation 1 …

Latex products

  • Patient with new job as a medical assistant (where she likely needs to where latex gloves) has developed a new rash on her hands, most consistent with allergic contact dermatitis
  • Key idea: Latex allergy seen in 1-5% of population and is seen at higher rates closer to 10% in individuals with chronic occupational exposure to latex (medical workers, etc.)

15. A previously healthy 16-year old boy is brought to the…

Viral infection

  • Patient presents with signs of encephalitis (altered mental status, emotional lability, fever, etc.) with CSF findings consistent with a viral infection (elevated WBC count with lymphocytic predominance, elevated protein and normal glucose), most consistent with HSV encephalitis
  • Fungal infection CSF: Elevated WBC count with lymphocytic predominance with decreased glucose
  • Bacterial infection CSF: Highly elevated WBC count (>1000) with neutrophilic predominance with decreased glucose
  • Normal CSF findings: WBC 0-5, Glucose 40-70 (2/3 serum glucose levels), Protein < 40
  • Key idea: Patients with suspected HSV encephalitis should be started on empiric IV acyclovir while awaiting PCR results and/or before brain MRI

16. A 52-year-old woman comes to the physician because of…

Adjustment disorder with anxiety

  • Patient with psychosocial distress causing impairment following a discrete stressor who does not meet criteria for more serious psychiatric diagnoses
  • Key idea: Adjustment disorder develops within 3 months of identifiable stressor and should last no longer than 6 months
  • Key idea: Treated with psychotherapy (CBT) +/- adjunctive pharmacotherapy for anxiety, insomnia, etc.

17. An 8-year-old girl with type 1 diabetes mellitus is brought to…

Infusion of 0.9% saline

  • Young patient involved in a motor vehicle accident who initially developed thigh pain and now has impaired consciousness with signs of hemodynamic instability most consistent with traumatic bleeding into the thighs, leading to hypovolemia
  • Key idea: Hematocrit can be normal during acute hemorrhage due to loss of equal parts of red cells and plasma, and may not drop until patient has received volume resuscitation
  • Key idea: Potential sites of bleeding in setting of traumatic hemorrhagic shock that can accommodate lots of bleeding are “Blood on the floor and 4 more”: (1) Floor (obvious external bleeding) (2) Chest (3) Abdomen (4) Pelvis (5) Thigh

18. A 52-year-old Cantonese woman comes to the physician…

Share the results with the husband only

  • Patients have the autonomy to decide whether or not they want to hear results of their own medical tests

19. A 21-year-old college student comes to the physician because…

Topical retinoic acid

  • Acne treatment in order of increasing severity/potency: Topical retinoids +/- benzoyl peroxide —> Topical antibiotics —> Oral antibiotics —> Oral isotretinoin
  • No good evidence that restricting milk products or sugar reduces acne

20. A 67-year-old woman has had anorexia, fatigue, and weakness…

Multiple myeloma

  • Multiple myeloma = Bone pain (often in the back) + Lytic bone changes on imaging + Anemia (due to plasma cells crowding out other hematopoeitic cells in bone marrow) + Elevated calcium concentration (due to lytic bone lesions) + Rouleaux formation on peripheral smear (due to sticky immunoglobulin proteins causing RBCs to stack up)
  • Key idea: If you see a patient with signs of kidney disease (elevated creatinine/BUN) and hypercalcemia, you should have a high index of suspicion for multiple myeloma because most other forms of kidney disease lead to hypocalcemia (due to binding of phosphate to calcium and reduced vitamin D activation)

21. A 32-year-old woman is brought to the emergency department…

Hemothorax

  • Opacities involving an entire lung lobe most consistent with collapsed lung tissue (atelectasis) vs. accumulation of blood/fluid/pus within that space
  • In this patient involved in a traumatic situation + signs of hemodynamic instability (tachycardic, borderline hypotensive) with complete opacification of right hemithorax and absent breath sounds on right most consistent with hemothorax
  • Pneumothorax would lead to lung lucency (increased air), diaphragmatic hernia almost always left sided (due to liver protecting right diaphragm) and ruptured bronchus often leads to persistent pneumothorax and/or pneumomediastinum

23. A 42-year-old man comes to the physician because of progressive swelling…

Constrictive pericarditis

  • Constrictive pericarditis = Patient with history of chest radiation + right-sided heart failure + Pericardial knock (early diastolic sound that can mimic S3 resulting from loss of pericardial elasticity during ventricular filling) + Kussmaul’s sign
  • Common causes of constrictive pericarditis include radiation therapy, cardiac surgery, connective tissue disorder, tuberculosis and viral pericarditis
  • Jugular venous distention that increases with inspiration = Kussmaul’s sign = Constrictive pericarditis, right heart failure/infarction, restrictive cardiomyopathy
  • Key idea: Patients with constrictive pericarditis or other forms of global diastolic dysfunction (restrictive cardiomyopathy) more commonly present with signs of right heart failure because the right heart is responsible for filling whereas the left heart is responsible for squeezing
  • Cor pulmonale = Right-sided heart failure due to primary lung disease (COPD, ILD, etc.)

24. A 35-year-old man is brought to the emergency department because…

Na 140 / Cl 80 / K 2.5 / HCO3 40

  • Vomiting classically leads to a hypochloremic, hypokalemic metabolic alkalosis
  • Key idea: Hypochloremia and metabolic alkalosis due to loss of HCl (stomach acid) whereas hypokalemia (and a small component of metabolic alkalosis) due to contraction alkalosis phenomenon in the collecting duct (reabsorbing Na and H20 in exchange for H+ and K+)

25. A previously healthy 52-year-old man comes to the emergency department…

Fluid restriction

  • Patient with significant smoking history presents with right hilar mass + hyponatremia = SIADH due to small cell lung cancer
  • Note: Hiccups could either be secondary to hyponatremia or could be due to lung mass irritating the diaphragm
  • Mild symptoms of SIADH (sodium >120, lethargy): Fluid restriction +/- salt tablets
  • Severe symptoms of SIADH (sodium <120, seizures, coma): Hypertonic (3%) saline
  • Key idea: Don’t want to correct hyponatremia too quickly in order to avoid osmotic demyelination syndrome (“low to high, the pons will die”)

26. A 27-year-old woman comes to the physician because of…

Irritable bowel syndrome

  • Young, otherwise healthy patient with intermittent constipation and intermittent diarrhea with no inflammatory/systemic signs (fever, weight loss, etc.) and normal exam
  • Key idea: Patients with IBS can have foul-smelling, mucus-coated stools
  • Key idea: Features inconsistent with IBS include rectal bleeding, nocturnal abdominal pain, weight loss, grossly abnormal lab findings (anemia, etc.), and older age

27. A 62-year-old woman comes to the physician because of generalized…

Avoidance of analgesics

  • Analgesic nephropathy (chronic interstitial nephropathy): Patient with chronic osteoarthritis treated with NSAIDs (and tylenol) who presents with signs of renal insufficiency (elevated creatinine and BUN) with mild proteinuria and sterile pyuria

28. A 37-year-old woman comes to the physician because of an itchy…

Medication adverse effect

  • Presentation most consistent with acute interstitial nephritis, which leads to combination of rash, AKI, pyuria, hematuria, WBC casts, eosinophilia and urinary eosinophils
  • Almost always caused by medication side effect, with common culprits being the 5 P’s (PPIs, Pain-free (NSAIDS), Pee (Diuretics), Penicillins and other antibiotics (especially sulfonamides such as in this patient) and rifamPin

29. An 87-year-old nursing home resident with dementia, Alzheimer’s…

Decreased gag reflex

  • Patient’s with Alzheimer’s disease are very prone to aspiration pneumonia both due to dysphagia and problems protecting their airway due to decreased gag reflex and impaired consciousness
  • Clues to aspiration pneumonia = Older patient (often demented) who presents with pneumonia in the right lung base (most common location due to right mainstem bronchus being wider, shorter and more vertical)
  • https://www.ncbi.nlm.nih.gov/pubmed/14577062

30. A 13-year-old girl is brought for a well-child examination…

Discussion of pregnancy prevention

  • Patient has normal sexual development and is not sexually active, so no reason for any of the lab tests or procedures
  • Important to discuss pregnancy prevention in adolescents, even in patients who are not yet sexually active

31. A 57-year-old woman with breast cancer comes to the physician…

Spinal cord decompression and cervical stabilization

  • Patient has signs/symptoms concerning for cervical spinal cord compression (neck pain with tenderness upon palpation, diffuse hyperreflexia) secondary to spinal metastases
  • Key idea: Acute spinal cord compression is a neurological emergency and requires immediate surgical decompression

32. A 5-week-old boy is brought to the physician because…

Hypertrophic pyloric stenosis

  • Classically leads to non-bilious (stenosis occurs above the duodenum) projectile vomiting in a newborn at 2-6 weeks of age that is refractory to dietary changes
  • Key idea: In questions asking about management, you want to first correct fluid and electrolyte abnormalities before proceeding with a corrective pyloromyomectomy

33. A previously healthy 44-year-old woman, gravida 4, para 4…

Stress incontinence

  • Classically leads to urine leakage with increased intra-abdominal pressure due to sneezing, coughing, laughing, and exercising
  • Risk factors for stress incontinence include obesity and multiple vaginal deliveries (as is the case with this patient)
  • Key idea: All types of urethroceles are associated with stress incontinence because urethroceles can disrupt the normal positioning/function of urinary sphincters

34. One day after an uncomplicated spontaneous vaginal delivery…

Overflow incontinence

  • Classic presentation of postpartum urinary retention (inability to void or small volume voids for at least 6 hours after vaginal delivery with incomplete emptying of bladder) complicated by overflow incontinence (dribbling of small amounts of urine)
  • Key idea: Post-void residual > 150 mL is a sign of urinary retention
  • Key idea: This specific presentation managed by intermittent catheterization

35. A 32-year-old woman, gravida 2, para 2, comes to the physician …

Anovulation

  • Differential for secondary amenorrhea = Pregnancy, Menopause, PCOS, hypothyroidism, prolactinoma, primary ovarian insufficiency, Asherman syndrome, functional hypothalamic amenorrhea, etc.
  • First step in setting of secondary amenorrhea (amenorrhea for >3 months in woman with previously regular menses or >6 months in woman with previously irregular menses) is to obtain a pregnancy test
  • Presence of withdrawal bleed after progesterone administration confirms that the uterus is undergoing a normal proliferative phase and therefore eliminates estrogen deficiency (primary ovarian failure, menopause, hyperprolactinemia, hypothyroidism, functional hypothalamic amenorrhea), endometrial abnormalities (Asherman’s) and Outlet tract abnormalities (imperforate hymen) as the cause of secondary amenorrhea
  • Therefore, secondary amenorrhea with progesterone withdrawal bleed indicates that the cause is anovulation, which is most commonly caused by PCOS

36. A healthy 21-year-old college student comes for an annual…

Pap smear

37. An asymptomatic 47-year-old man comes for a preemployment…

Serum lipid studies while fasting

  • Key idea: Abnormal total cholesterol levels should always be followed by fasting LDL levels because treatment is based on LDL and HDL levels/thresholds

38. Four weeks after a low transverse cesarean delivery…

Reassurance

  • Key idea: Pulling effect/feeling is common complaint in the post-partum for patients who underwent C-section and is due to healing fibrosis around the wound site
  • No fever, wound erythema/purulence, or feeling of hematoma/seroma beneath the wound site
  • Note: Women are often advised to avoid exercise or sexual activity for at a minimum of 6 weeks after C-section

39. A 23-year old woman has pain, cramping, and swelling of the…

Hypercoagulable state of pregnancy

  • Presentation is most consistent with a Deep Vein Thrombosis, which is a feared complication of pregnancy
  • Key idea: Pregnancy is a hypercoagulable state, which likely evolved because we don’t want women bleeding out during delivery

40. A 62-year-old man has had the gradual onset…

Tricuspid regurgitation

  • Murmur that is loudest on inspiration = Right-sided valvular defect (tricuspid or pulmonic) —> Location of murmur = Tricuspid valve —> Holosystolic tricuspid valve murmur = Tricuspid regurgitation
  • Large wave occurring with S2 caused by blood aberrantly moving from RV to RA and up into SVC during systole
  • Eventually leads to RV hypertrophy due to increased preload (lifting systolic murmur of the sternum) and eventually right heart failure (congestive hepatomegaly, lower extremity edema, etc.)

42. A 25-year-old woman comes to the emergency department…

Idiopathic intracranial hypertension

  • Idiopathic intracranial hypertension = Pseudotumor cerebri with risk factors being Female TOAD (Female, Tetracyclines, Obesity, vitamin A and Danazol)
  • Often presents in young overweight women with combination of headaches, vision loss (enlarged blind spot), and pulsatile tinnitus with signs of increased intracranial pressure (CN 6 palsy, papilledema, lumbar puncture findings of increased opening pressure [>250])
  • Key idea: Treat with weight loss and acetazolamide

43. A 25-year-old man with a history of intravenous drug use…

Cryptococcal meningitis

  • Patient with risk factors for immunosuppression (IVDU can lead to HIV due to needle sharing) presents with signs of increased intracranial pressure (papilledema and weakness of lateral rectus muscle [CN 6 palsy]) and meningitis (headache, fever, neck stiffness) with CSF findings consistent with a fungal etiology (elevated WBC count with lymphocytic predominance, low glucose)
  • Key idea: Cryptococcal meningitis classically leads to meningitis symptoms + signs of elevated ICP + umbilicated skin lesions
  • Note: Elevated ICP preferentially leads to a CN 6 palsy is a contested issue with two potential explanations: (1) It has the longest intracranial course and therefore is prone to stretching in setting of elevated ICP (2) CN 6 emerges straight from brain stem rather than obliquely/transversely like other cranial nerves, and is therefore more prone to compression by backward brain displacement in setting of elevated ICP

44. A 7-year-old girl is brought to the physician in September…

Throat culture

  • Pharyngitis and fever without viral URI symptoms in a young child always concerning for Strep throat, which requires rapid strep test followed by throat culture if negative [NO EMPIRIC TREATMENT IN KIDS]
  • Key idea: In adults, patients risk of Strep throat is stratified based on Centor criteria [(1) Fever by history (2) Tender anterior cervical lymphadenopathy (3) Tonsillar exudates (4) Absence of cough], and with patients receiving no testing (scores 0-1), rapid strep test followed by throat culture if negative (2-3) or empiric antibiotics (4+)
  • Key idea: In contrast, CENTOR criteria are not applied to children, with children who have potential signs of strep throat (exudative pharyngitis, palatal petechiae) and NO viral symptoms (cough, rhinorrhea, conjunctivitis, oral ulcers) receiving a rapid antigen test followed by throat culture if rapid strep test is negative (i.e. all children receive diagnostic tests and none are treated empirically)

45. After an uncomplicated laparoscopic cholecystectomy, a…

Reinsertion of a Foley catheter

  • Presentation most consistent with acute urinary retention given demographic features (older man who is post-op) who presents with suprapubic tenderness (abdominal pain) and anuria unresponsive to fluids
  • Managed via catheter insertion and urinalysis (rule out UTI)
  • Patient is unresponsive to 1 L, so unlikely that another 1 L will help
  • Patient is producing no urine, so diuretic is unlikely to help

46. A 20-year-old man is brought to the emergency department…

Inadequate dissipation of body heat

  • Presentation most consistent with heat stroke, which leads to elevated temperature and CNS dysfunction +/- other organ damage in a patient participating in strenuous activity in hot/humid weather and is caused by impaired thermoregulation
  • Key idea: Managed with aggressive cooling and fluid/electrolyte correction with NO role for antipyretic therapy
  • Contrast with heat exhaustion, which leads to elevated temperature with no CNS dysfunction and is caused by inadequate salt and water replacement

Exam section 3:

2. An 18-year-old man comes for an examination prior to participation…

Germinal cell tumor

  • Presentation most consistent with testicular cancer, with germ cell tumor being the most common cause
  • Testicular mass that does not transiluminate = Cancer or varicocele
  • Varicocele would not lead to a discrete mass, but rather a “bag of worms” feeling and it would decrease in size when patient placed in supine position due to increased venous return to heart
  • Testicular mass that does transiluminate = hydrocele or spermatocele
  • Key idea: Oftentimes patients discover masses after trauma to that area because they are paying more attention to the area, so don’t automatically write-off the mass as being the result of trauma (for example, many NBME questions have patients finding breast masses after trauma to area)

3. A 50-year-old woman has had progressive dyspnea over the past…

Echocardiography

  • Presentation is most consistent with cardiac tamponade (chest pain + hypotension + pulsus paradoxus + jugular venous distention + “water-bottle” heart on CXR) most likely due to breast cancer recurrence/metastasis to pericardium (2nd most common cause of malignant pericardial effusion)
  • Next best step would actually probably be pericardial window, but of the choices echocardiography is most appropriate
  • Pericardiocentesis: Drainage of pericardium
  • Thoracentesis: Drainage of pleural space
  • Paracentesis: Drainage of peritoneal cavity

4. A county health officer investigates an outbreak of illness…

Staphylococcus aureus

  • Causes of rapid-onset food poison (< 6 hours) are due to ingestion of pre-formed toxin and are most commonly caused by Staph aureus (cream-based food such as egg salad) or Bacillus cereus (reheated rice syndrome)
  • Clostridium perfringens (improperly cooked/stored meat) is another cause of food poisoning, but often occurs 6-24 hours after ingestion because patients ingest spores which need germinate in digestive tract to secrete toxin

5. A 38-year-old woman, gravida 2, para 1, at 38 weeks’…

Nonstress test

  • If a patient presents with decreased fetal movement, first perform a non-stress test to look at fetal heart rate (110-160 beats/min), variability (consistent and not too much, but not none at all) and acceleration (15/15 2 in 20: heart rate should rise by at least 15 beats/min for at least 15 seconds at least 2 times in 20 minutes)
  • If that test result is abnormal, then perform a non-stress test with acoustic stimulation and assess the same way you assess non-stress test (rate, variability and acceleration)
  • Abnormal non-stress test with stimulation means you should perform a biophysical profile, which is similar to APGAR and assigns a score from 0 to 2 in 5 categories (non-stress test result, amniotic fluid index, fetal breathing, fetal movements and fetal tone)
  • Unequivocal biophysical profile (greater than 2 but below 8) should be followed up with contraction stress test to look for late decelerations (decelerations begin at same time that contractions occur) and/or bradycardia (HR < 110 bpm)

6. A 52-year-old woman comes to the physician because of a 3-month…

Intraductal papilloma

  • Most common cause of unilateral bloody nipple discharge, and often presents without mass or lymphadenopathy

7. A 17-year-old boy is brought to the emergency department by…

PCP

  • PCP intoxication characterized by agitation, nystagmus and aggressive behavior

8. A previously healthy 62-year-old man comes to the emergency…

CT scan of the abdomen

  • Presentation most consistent with diverticulitis, which classically leads to fever + LLQ pain in an older person /// First step in work-up is abdominal CT

9. A 47-year-old man comes to the physician because of fever and chills…

Carbamazepine therapy

  • WBC differential shows severe neutropenia even though patient likely has an active infection, which is most consistent with a diagnosis of agranulocytosis
  • 3 drug classes that are associated with agranulocytosis include (1) Carbamazepine (antiseizure) (2) Clozapine (antipsychotic) and (3) Methimazole/Propylthiouracil (anti-thyroid)
  • Key idea: Classic presentation of agranulocytosis is sore throat + fever + low neutrophil count
  • Note: Patient also has a mildly elevated MCV, which is likely secondary to significant alcohol exposure

10. A 62-year-old woman comes to the physician because of…

Malabsorption

  • Presentation most consistent with lactose intolerance (bloating, abdominal pain and intermittent diarrhea that have worsened with increased dairy intake) which is a form of malabsorption because lactose cannot be broken gown into glucose and galactose monomers
  • Impaired intestinal motility would lead to decreased bowel sounds
  • Inflammatory process would likely lead to fever +/- blood in stool

11. A 42-year-old man comes to the emergency department because…

Amphotericin B

  • Presentation is most consistent with Cryptococcal meningitis (headache, neck stiffness, fever, signs of elevated intracranial pressure, CSF consistent with fungal infection (low glucose with lymphocytic predominant WBC elevation) and they tell us that the cryptococcal antigen assay is positive
  • Patients are often treated with Amphotericin + Flucytosine initially for about 2 weeks and then switched over to fluconazole for another few months

12. Two days after admission to the hospital for treatment of severe…

Acute tubular necrosis

  • Patient presented with hemodynamic instability secondary to sepsis and developed an AKI (creatinine concentration increased by at least 0.3 mg/dL)
  • Two most likely causes of AKI given hypotension are pre-renal vs. Acute tubular necrosis
  • Features that point to ATN in this case include BUN:Cr ratio < 15, FeNa > 2% and muddy brown casts on urinalysis
  • In contrast, pre-renal AKI would lead to BUN:Cr ratio > 20 and a FeNa < 1% (both due to increased reabsorption of Na+ from renal tubules in order to compensate for hypovolemia)

13. A 77-year-old woman is brought to the physician…

Memory loss

  • NBME loves to ask about complaints in elderly patients to assess whether you know which changes are physiological vs. pathological (same with newborns and developmental milestones)
  • In this patient, all of the findings are normal except for the memory loss because it has only been present for 1 month, which does not correspond with the typical picture of Alzheimer’s
  • Patients with suspected cognitive impairment are often worked up with cognitive tests (MOCA, MMSE, Mini-Cog), Lab testing (CBC, B12, TSH, BMP +/- syphilis) and Imaging (CT or MRI of brain)
  • Key idea: Mild cognitive impairment = short-term memory/cognitive problems that are concerning to patient/family but that DON’T affect daily life or ADLs (using phone, driving, cooking, etc.)

14. A 2-year-old boy is brought to the physician because of fever…

Intravenous immunoglobulin infusion

  • Patient has an X-linked condition (2 maternal uncles affected) that leads to recurrent infection with encapsulated bacteria (Strep pneumoniae) with labs showing normal lymphocyte count with very low immunoglobulins, most consistent with X-linked (Bruton’s) agammaglobulinemia
  • Patient should receive treatment of acute infection and IV immunoglobulin infusion in order to make up for lack of endogenous immunoglobulin production

15. A 67-year-old man with long-standing signs and symptoms of…

Protein 2.5 / Glucose 90 / Leukocyte count 2000 / Segmented neutrophils 60% / Monocytes 40%

  • Pleural effusion is most likely transudative, so we would expect low protein, relatively low leukocyte count and a glucose near normal serum levels, which is most consistent with selected answer
  • Key idea: For new pleural effusions, often can determine transudative vs. exudative using Light’s criteria where if they have at least 1 of 3 positive then it is exudative: (1) Pleural protein:Serum protein > 0.5 (2) Pleural LDH:Serum LDH > 0.6 (3) Pleural LDH > 2/3 normal serum LDH levels

16. A 57-year old man is brought to the emergency department…

Carotid ultrasonography

  • Patient with vasculopathic risk factors (hypertension and poorly controlled diabetes) who presents with recurrent embolic phenomenon to brain all from the same side (will lead to ipsilateral visual problems (amaurosis fugax) and contralateral weakness (due to MCA involvement))
  • Key idea: Amaurosis fugax is highly associated with carotid artery disease

17. A previously healthy 27-year-old woman is brought to the …

Electromyography and nerve conduction studies

  • Presentation most consistent with Guillain-Barre syndrome (development of ascending weakness, decreased reflexes, and sensory symptoms over the course of days-weeks in a young patient with a recent URI or GI illness)
  • Best tests to confirm Guillain-Barre are lumbar puncture (showing increased protein with normal leukocytes (albuminocytologic dissociation) or electromyography/nerve conduction studies because the underlying pathophysiology is a demyelinating disease

18. A previously healthy 42-year-old carpenter comes to the…

Median nerve at the wrist

  • Distribution most consistent with median nerve, and the history of patient with recurrent use of hands (carpenter) with pain and numbness at night most consistent with carpal tunnel syndrome, which is due to compression of median nerve at the wrist
  • Key idea: Carpal tunnel syndrome leads to spared sensation to the palm because the palmar cutaneous branch enters the hand external to the carpal tunnel
https://www.anatomynote.com/human-anatomy/nerves-system/median-nerve-ulnar-nerve-radial-nerve-innervation-area-in-hand/

19. A previously healthy 37-year-old man comes to the physician…

Ulnar nerve at the elbow

  • Patient has numbness and weakness in the distribution of the 5th finger and 1/2 of the 4th finger, which is consistent with the ulnar nerve
  • Key idea: Most common site of ulnar nerve compression is at the elbow because it can be compressed when the elbows are resting while sitting at a desk, driving, etc., and is especially relevant in this patient who has paresthesias with compression of the cubital tunnel (which is where ulnar nerve passes by the elbow)
https://teachmeanatomy.info/upper-limb/nerves/ulnar-nerve/

20. An 18-year-old man comes to the physician for an initial…

Ask about suicidal feelings

  • Patient has presentation consistent with somatic symptom disorder, although this patient also has features of psychosocial distress (hopeless, fatigue, etc.) concerning for depression with somatic symptoms
  • Key idea: In patients with depression, always ask about suicidal ideation/plan/intent

21. A previously healthy 67-year-old man is admitted to the hospital…

Small cell carcinoma

  • Patient who presents with non-specific lethargy, confusion and muscle cramps found to have severe hyponatremia with a lung mass most consistent with Small cell carcinoma complicated by SIADH

22. The genetic disease institute at a university hospital…

  • If we decrease the prevalence of a disease in a population, then the ratio of true positives:false positives tips towards false positives (we would expect fewer true positives), leading to reduced PPV
  • At the same time, a decrease in disease prevalence causes the ratio of true negatives:false negatives to tip towards true negatives (we would expect more true negatives), leading to increased NPV
  • Key idea: Sensitivity and specificity are not affected by disease prevalence (characteristics of test)

23. An asymptomatic 32-year old woman comes for a routine…

Weight-loss program

  • Most important risk factor for back pain and osteoarthritis is obesity
  • Answer would not be limiting physical activity because patients with acute back pain are actually advised to remain active and to avoid bed rest

24. A 32-year-old man receiving intensive chemotherapy for…

Pneumocystic jiroveci (formerly P. carinii)

  • Immunocompromised patient (intensive chemotherapy) develops fever with diffuse reticular opacities on chest x-ray most consistent with PCP
  • Key idea: Patients often have elevated LDH levels on lab testing and treatment consists of TMP-SMX +/- prednisone (if patient is hypoxic)

25. A 64-year-old woman has moderately severe postoperative pain…

Patient-controlled intravenous morphine

  • Patient is in severe pain after a major surgery, and therefore would benefit from use of opiate medications
  • A common form of pain control in the hospital is placing patients on patient-controlled IV morphine where the patient is allowed to self-administer doses, but the physician team can put limits as to the frequency and total amounts of morphine that are administered
  • Aspirin-codeine is more appropriate for mild-moderate pain, and transcutaneous fentanyl patches are a form of long-acting opiates that are not used in the post-op patient but rather in patients with chronic pain needs (example: patient with metastatic cancer with painful bony mets)

26. Three days after being hospitalized for treatment of a hip…

Heparin therapy

  • Post-op patient who has been bed-bound for multiple days has developed dyspnea and hemoptysis with V/Q scan showing areas of mismatch, most consistent with acute pulmonary embolism
  • Key idea: The first step in management of acute PE is administration of an anticoagulant, most often heparin (assuming patient has no absolute contraindications such as active bleeding, hemorrhagic stroke, etc.)
  • Some patients who are hemodynamically unstable (which is not the case here) will also receive thrombolysis either through use of fibrinolytics or interventional radiology procedures, but even these patients are often first started on a heparin drip while definitive diagnostic tests (such as CT-PE) are performed
https://www.uptodate.com/contents/image?imageKey=PULM%2F57249~PULM%2F99762&topicKey=PULM%2F8265&source=see_link

27. A 32-year-old woman comes to the physician because of …

Clue cells

  • Patient has grayish vaginal discharge with pH > 4.5, which is most consistent with bacterial vaginosis, and therefore we would expect clue cells
  • 3 main forms of vaginal infection and definitive features include (1) Bacterial vaginosis [gray fishy discharge, pH > 4.5, no cervical/vaginal erythema, clue cells, treat with metronidazole or clindamycin] (2) Candidal vaginitis [white cottage cheese discharge, pH < 4.5, cervical/vaginal erythema, pseudohyphae and budding yeast, treat with oral or topical -azole drugs] (3) Trichomonal vaginitis [greenish-yellowish discharge, pH > 4.5, cervical/vaginal erythema, motile trichomonads, treat with metronidazole

28. A previously healthy 67-year-old man comes to the physician…

Daily exercise program

  • Key idea: Patients with claudication (such as this patient who has significant vasculopathic risk factors and has leg pain with exercise) should first be optimized with smoking cessation + daily exercise program + statin + aspirin
  • Key idea: Decision to pursue revascularization in stting of peripheral vascular disease is NOT dependent on ABI cut-off and instead is indicated in setting of (1) Rest pain (2) Ischemic ulceration (arterial ulcer) (3) Gangrene
  • Claudication: Patient will have risk factors for peripheral vascular disease (diabetes, HTN, smoking, etc.), reduced lower extremity pulses, reduced lower extrmity temperature, pain classically in the calves, reduced hair on legs
  • Pseudoclaudication (spinal stenosis): Positional (improves with flexion), classically affects buttocks and thighs, may be associated with back pain

29. A 67-year-old man comes to the physician because of a …

Thymoma

  • Presentation is most consistent with myasthenia gravis (tends to affect young women and older men and classically leads to dysphagia, dysarthria and eye weakness that is worse at the end of the day)
  • Key idea: Patients with myasthenia gravis should be treated with an acetylcholinesterase inhibitor (most often pyridostigmine), but they should also have a CT scan performed because in about 15% of cases the cause is a thymoma and some patients will be cured following removal of the thymoma
  • Note: If patient with weakness is described in vignette and they give you some sort of chest imaging, you should have high index of suspicion for myasthenia gravis (associated with thymoma) or Lambert-Eaton (associated with small cell lung cancer)
https://www.pennmedicine.org/for-health-care-professionals/for-physicians/physician-education-and-resources/clinical-briefings/2019/march/medical-and-surgical-management-of-myasthenia-gravis-and-thymoma

30. A 42-year-old woman comes to the physician because of right…

Endoscopic retrograde cholangiopancreatography

  • Patient who had gallbladder removed 2 months ago presents with right-sided abdominal pain, clinical jaundice, cholestatic pattern of LFTs (increased direct bilirubin and increased alkaline phosphatase) along with dilated intrahepatic biliary ducts, most consistent with a retained stone within the common bile duct that needs to be investigated and treated by ERCP
  • Key idea: Although patient had gallbladder removed, patients can sometimes have residual stones within the gallbladder remnant, cystic duct or common bile duct that remain behind and can cause pathology
  • Key idea: ERCP indicated in setting of choledocolithiasis because of its association with acute cholangitis and the morbidity/mortality associated with that disease
  • Note: HIDA scan (cholescintigraphy) is often not the answer on NBME exams, with its main indication being in a patient with suspected acute cholecystitis who had negative findings on RUQ ultrasound

31. A 37-year-old woman, gravida 2, para 1, at 12 weeks’ gestation…

Previous preterm delivery

  • The two most important risk factors for preterm labor include (1) History of previous preterm labor (2) Short cervical length (<2 cm in patient without history of preterm labor or <2.5 cm in patient with history of preterm labor)

32. A previously healthy 16-year-old high school wrestler comes…

Eczema herpeticum

  • Patient with baseline atopic dermatitis (atopic patient [allergic rhinitis] with erythema and lichenification over antecubital and popliteal fossae) who presents with acute painful, umbilicated vesicles over sites of atopic dermatitis is most consistent with eczema herpeticum caused by HSV-1
  • Note: Patient is a wrestler, which in NBME language means they have high risk of being exposed to skin infection because they are rolling around on sweaty mats
  • Key idea: Four infectious complications of atopic dermatitis include (1) Eczema herpeticum [HSV-1, painful vesicular rash] (2) Impetigo (Staph or Strep, painful pustules with honey-colored crust) (3) Molluscum contagiosum (Poxvirus, umbilicated flesh-colored papules) (4) Tinea corporis (Tricophyton, pruritic patch with central clearing and raised border with overlying scale)

33. A 42-year-old woman comes to the physician for evaluation of persistently…

Autonomous production of aldosterone

  • Key idea: Young, otherwise healthy women should not present with severe essential hypertension, and it should make you have a high index of suspicion for secondary hypertension (fibromuscular dysplasia, hyperaldosteronism, coarctation of the aorta, Cushing’s syndrome, hypo/hyperthyroidism, pheochromocytoma, etc.)
  • In this patient, her labs demonstrate a hypokalemic metabolic alkalosis, which is consistent with a state of elevated RAAS activity either secondary to poor renal perfusion (question would tell you a bruit is present) or endogenous, inappropriate production of aldosterone

34. A 67-year-old man has had shortness of breath on exertion…

Thoracentesis

  • Older patient with significant smoking history and subacute weight loss who appears chronically ill presents with a large left-sided pleural effusion and should therefore receive thoracentesis in order to determine the etiology of the effusion (which is most likely malignant)
  • Note: Only reason to not get a thoracentesis in the setting of a new pleural effusion is if you have very high index of suspicion for heart failure and the patient’s effusion responds well to diuretic therapy
  • Key idea: Pleural effusion with malignant cells seen on cytology in setting of lung cancer consistent with Stage 4 disease because it is caused by metastasis of lung cancer to the pleura and not by pleural fluid production triggered by the primary lung cancer
  • Key idea: When trying to decide where to sample/biopsy in setting of suspected cancer, one of the considerations (in addition to risk of procedure and access to site) includes trying to select the site that would enable the most up-staging of the cancer because it has important prognostic and therapeutic implications; in this case, if the pleural fluid is found to have malignant lung cancer cells, then the patient has automatic Stage 4 disease

35. A 26-year-old woman is brought to the emergency department…

ACTH stimulation test

  • Young female patient with autoimmune history presents with fatigue, weakness, abdominal complaints and generalized hyperpigmentation found to have eosinophilia, hyponatremia, hyperkalemia and metabolic acidosis on labs, most consistent with Addison’s disease which should be worked up with an ACTH stimulation test (would typically lead to increased cortisol levels in healthy patient but will lead to a less robust response in patient with Addison’s disease)
  • Key idea: High cortisol/steroid states lead to increased neutrophil count, decreased lymphocyte count and decreased eosinophil count, so patients with Addison’s disease have the opposite
  • Key idea: Addison’s disease leads to primary failure of adrenal gland, such that the aldosterone-producing cells are also affected, leading to the hyperkalemia, hyponatremia and metabolic acidosis because aldosterone typically works at collecting tubule to reabsorb sodium in exchange for potassium and protons
  • Key idea: Hyperpigmentation (in addition to hyperkalemia) helps to distinguish primary from secondary/tertiary adrenal failure because in primary adrenal failure (Addison’s) the pathology is in cortisol production by the adrenal, so there will be less negative feedback of cortisol on the pituitary, leading to increased production of ACTH, with increased production of ACTH also leading to increased production of melanocortin because they share the same precursor protein, with melanocortin leading to hyperpigmentation [contrast with secondary/tertiarty adrenal failure where ACTH is low]

36. A 40-year-old man is brought to the emergency department…

Myocardial contusion

  • Patient involved in a motor vehicle accident presents with severe chest pain, hypotension and arrhythmia and decompensates further with administration of fluids (because patient is basically in cardiogenic shock and cannot handle increased volume), most consistent with myocardial contusion in setting of blunt cardiac injury
  • Myocardial infarction would lead to ST-segment changes, pulmonary contusion would primarily lead to respiratory symptoms and traumatic rupture of the aorta would lead to severe hemodynamic instability and likely a positive response to fluids (although most of these patients die in the field due to the high volume of blood loss they experience)

37. A 32-year old man with alcoholism is brought to the emergency…

Vitamin B1 (thiamine)

  • Patient with chronic alcoholism present with confusion, ataxia and ophthalmoplegia, most consistent with Wernicke encephalopathy secondary to thiamine deficiency
  • Key idea: On the NBME exam, patient with alcoholism = patient with malnutrition and vitamin deficiencies (among other associations)

38. A 72-year-old man with hypertension has had increasingly severe…

Metastatic prostate carcinoma

  • Older man with back pain, tenderness over spine, and osteoblastic vertebral lesions most consistent with metastatic prostate cancer
  • Mnemonic for causes of osteoblastic bone metastases: Holster, Point, Shoot and Blast the enemy – Hodgkin lymphoma, Prostate cancer, Small cell lung cancer lead go osteoblastic lesions

39. A 6-month-old girl is brought to the physician for a …

Obstruction of cerebrospinal fluid flow

  • Question stem tells us that patient has increased intracranial pressure secondary to hydrocephalus (increasing head circumference with bulging fontanelles)
  • Epidemiologically the most common cause of hydrocephalus in an infant is aqueductal stenosis (obstruction of CSF flow from the 3rd ventricle to the 4th ventricle) and this patient may have additional CSF flow obstruction secondary to an Arnold Chiari type 2 malformation (associated with myelomeningocele)
  • Arnold-Chiari type 1 malformation associated with syringomyelia, whereas Arnold-Chiari type 2 malformation associated with myelomeningocele (with this patient has)

40. A 47-year old man is admitted to the hospital after threatening…

Haloperidol decanoate

  • Patient has a presentation consistent with schizophrenia that is responsive to antipsychotic treatment, but with multiple relapses due to poor medication adherence
  • Patients with unstable psychiatric illness who have frequent medication non-adherance, poor insight or little social support are good candidates for long-acting injectable antipsychotic, with one of the common formulations being haloperidol decanoate
  • Contrast with clozapine, which is indicated in setting of treatment-resistant schizophrenia and schizophrenia with suicidality

41. A newborn is in severe respiratory distress immediately …

Mother: O, Rh-negative / Newborn: O, Rh-positive

  • Newborn born to mother without prenatal care who has previously been pregnant (G2P1) is found to have anasarca (diffuse body edema) with significant edema and positive direct Coombs’ test, most consistent with Rh hemolytic disease
  • Key idea: Anasarca/hydrops fetalis occurs due to high output heart failure secondary to profound anemia seen in Rh hemolytic disease
  • Key idea: ABO incompatibility will not present with as severe of a presentation because majority of maternal antibodies against other AABO blood groups are IgM and do not readily cross the placenta (contrast with acquired Rh-targeting antibodies in Rh-negative mother who has been exposed to Rh-positive blood which are IgG and can easily cross the placenta)
  • Potential causes of anasarca or hydrops fetalis: Rh-hemolytic disease, parvovirus/CMV infection of mother, hemoglobin Barts disease (severe form of alpha thalassemia), etc.

42. A 19-year-old man is brought to the emergency department 45…

Laparotomy

  • Indications for urgent exploratory laparotomy (ex-lap) in setting of penetrating abdominal trauma includes (1) Hemodynamic instability (which this patient has) (2) Peritonitis (rebound, guarding) (3) Evisceration (externally exposed organs) (4) Blood from NG tube or on rectal exam
  • Note: If patient has no indications for immediate ex-lap, then next step would be local exploration of the wound and an extended FAST scan

43. A 52-year-old man comes to the physician for a routine health…

Exercise stress test

  • 2 key indications for an exercise stress test include (1) History compatible with coronary artery disease (angina, etc.) (2) Risk stratification prior to starting exercise plan
  • Spirometry would be used to evaluated dyspnea and cardiac catheterization would only be used if patient had positive exercise stress test

44. A 32-year-old woman who is HIV positive has a CD4+…

  • All patients (HIV or not) should receive the annual Influenza vaccine and the Tdap every 10 years
  • Additional vaccination requirements for patients with HIV include (1) Vaccination for hepatitis B unless they have documented immunity (2) Strep Pneumo PCV13 followed by the 23-valent PPSV23 8 weeks later and again in 5 years and at age 65 (3) Meningococcal vaccine with boosters every 5 years

45. A 16-year-old girl is brought to the physician because of episodes…

Aortic incompetence

  • Patient with Marfanoid habitus (tall, long arms, long fingers) presents with heart symptoms and is found to have bounding peripheral pulses (consistent with widened pulse pressure) and an early diastolic murmur without clear localization most consistent with aortic regurgitationt/incompetence
  • Key idea: Patient’s with Marfan’s syndrome are prone to aortic aneurysm, with a thoracic aneurysm leading to widening of the aortic valve ring with resulting aortic incompetence
  • Note: Localization of aortic regurgitation murmur can sometimes help to detemine the location of the pathology, with loudest murmur at right sternal border most consistent with aortic root disease and loudest murmur at left sternal border most consistent with primary valvular disease

46. A 3-year-old boy is brought for a follow-up examination…

CT scan of the head

  • Presentation most consistent with mastoiditis (patient with acute otitis media who develops pain behind the ear with displacement of the auricle
  • Patients with mastoiditis should be worked-up with CT scan of the head in order to look for potential complications such as abscess that would affect management
  • Patients with pure mastoiditis without abscess: IV antibiotics
  • Patients with mastoiditis and abscess: IV antibiotics +/- tympanostomy or mastoidectomy

Exam section 4:

1. A study is conducted to compare the effectiveness of lorazepam…

Confounding variables

  • Patients in the lorazepam group were much more likely to receive a paralytic agent, which is an important confounder and calls into question the internal validity of the study
  • Key idea: Randomization is often performed in studies with the idea that it swill equally disperse potential confounders between the control and treatment arms of the study

2. A 70-year-old man comes to the physician because of…

Outflow obstruction of the bladder

  • Patient with chronic urinary hesitancy and frequency who is afebrile is found to have an enlarged nontender prostate and a positive urinalysis (pyuria and bacteria), most consistent with asymptomatic bacteriuria in the setting of BPH
  • Key idea: For men, BPH with bladder outlet obstruction is reported to be the major predisposing factor for the development of asymptomatic bacteriuria (positive urinalysis but asymptomatic) because it leads to urine stasis
  • Infection of epididymis would lead to unilateral posterior testicular pain improved with testicular elevation, infection of urethra would lead to UTI symptoms (dysuria, suprapubic tenderness), and acute prostatitis would lead to systemic symptoms (fever, chills, malaise)and tender prostate on DRE

3. A 15-year-old boy is brought to the emergency department…

Hypertrophic obstructive cardiomyopathy

  • Presentation of a young healthy child with exertional loss of consciousness without a prodrome found to have a systolic ejection murmur in the aortic/pulmonic valve distribution that becomes louder with decreased preload (upon standing) most consistent with HOCM
  • Key idea: While vasovagal syncope would be most common cause of LOC, it is preceded by prodrome of nausea, sweating, etc.
  • Key idea: Most murmurs improve with decreases in preload (upon standing), with the two exceptions being HOCM (decreased preload leads to less stretching of the LV and therefore more obstruction near aortic valve due to septal hypertrophy) and Mitral valve prolapse (less preload leads to less stretching of the LV, leading to less taut chordae tendinae which will be more prone to relaxation and enable earlier prolapse of the mitral valve leaflet)

4. A 62-year-old woman is brought to the emergency department 1 hour…

Orthostatic hypotension

  • Older patient recently started on anticoagulation therapy who has been passing dark bloody stools for 2 days presents with loss of consciousness upon standing found to have hypotension and tachycardia on exam, most consistent with orthostatic hypotension
  • Key idea: Orthostatic hypotension formally diagnosed if patient has drop in blood pressure by at least 20 systolic and/or at least 10 diastolic upon standing OR if patient has consistent light-headedness or LOC upon standing

5. A previously healthy 52-year-old woman comes to the physician…

Sporotrichosis

  • Sporothrix schenckii is a dimorphic fungus found in decaying plant matter and soil with can often lead to an ulcerative skin papule with proximal lesions along lymphatic chain in patients with exposure to plants (often in gardeners)

6. A 57-year-old man comes to the physician because of intermittent…

Overflow incontinence from acontractile bladder

  • Patient with history of poorly controlled T2DM with signs of autonomic neuropathy (gastroparesis) presents with intermittent urinary incontinence found to have a post-void residual volume of 500 mL most consistent with overflow incontinence secondary to poor bladder contraction
  • Key idea: Post-void residual > 150 mL is a sign of urinary retention
  • Key idea: Autonomic neuropathy can be seen in late-stage diabetes and can manifest as problems with esophageal motility (dysphagia), gastric emptying (gastroparesis), intestinal function (constipation), blood pressure (orthostatic blood pressure unresponsive to fluids) and bladder function (acontractile bladder, incontinence)

7. A 47-year-old woman comes to the physician for a follow-up…

Gastric bypass

  • Patient is a middle-aged woman with a BMI of 75, poorly controlled type 2 diabetes and significant limitations in activities of daily living due to obesity who is compliant with her diet and medications and has been unsuccessful at losing weight on her own who would be a candidate for gastric bypass
  • Indications for gastric bypass: Patient with multiple failed attempts to lose weight with adequate ability to follow-up and adhere to a plan who also has (1) BMI > 40 or (2) BMI >35 with obesity-related comorbidity (Type 2 diabetes, obstructive sleep apnea, obesity hypoventilation syndrome, debilitating osteoarthritis, cardiomyopathy, coronary artery disease)
  • Note: Interestingly, bariatric surgery in patients with comorbid type 2 diabetes has been found to significantly improve and even cure type 2 diabetes in a significant proportion of patients (https://care.diabetesjournals.org/content/34/Supplement_2/S361)

8. A 16-year-old boy with neurofibromatosis is brought for a …

Catecholamine-producing tumor

  • Young patient with neurofibromatosis who presents with episodes of headaches and flushing who is found to have elevated blood pressure, most consistent with pheochromocytoma
  • Neurofibromatosis Type 1: Cafe-au-lait spots, intellectual disability, cutaneous neurofibromas, Lisch nodules, optic glioma, Pheochromocytomas, seizures (often secondary to meningioma)
  • Neurofibromatosis type 2: Leads to bilateral (2) vestibular schawannomas, affects both (2) eyes (juvenile cataracts and leads to 2 other types of brain tumor (meningioma, ependymoma

9. A 17-year-old boy sustains a head injury and loses consciousness…

Acute respiratory distress syndrome

  • Patients who have a near-drowning event are still at risk for morbidity/mortality from delayed or secondary drowning hours to days later
  • The underlying pathophysiology is related to inflammation secondary to the aspiration of water into the lung, and the way that I remember the association between near-drowning and ARDS is that the inhaled water washes away the surfactant leading to ARDS, analogous to how preterm newborns can have neonatal respiratory distress syndrome due to low surfactant levels

10. Four days after undergoing resection of an obstructing sigmoid…

Central venous catheter

  • Most common types of hospital-acquired infections are central-line associated bloodstream infections (CLABSI) which often lead to Staph infection, Catheter-associated Urinary tract infections (CAUTI) which often lead to E. coli infection, Surgical Site Infection which often leads to Staph or Strep and Ventilator-associated pneumonia which leads to Strep pneumo, H. influenzae, Staph, etc. infections
  • Intra-abdominal abscess would most commonly lead to anaerobic, gram-negative infection and while operative wound could lead to Staph or Strep infection you would expect the colostomy to be erythematous, purulent, etc.

11. A 28-month-old boy has a history of cyanosis since birth with…

Tetralogy of Fallot

  • Infant/toddler with early cyanosis and clubbing with murmur consistent with RV hypertrophy (RV heave), Pulmonic stenosis (single S2), and a VSD (systolic murmur), which are 3/4 of the features of Tetralogy of Fallot
  • Key idea: Causes of early cyanosis from primary heart defects can be remembered with mnemonic 1, 2, 3, 4 and 5 because it can be caused by (1) persistent truncus arteriosus (1 vessel) (2) transposition of the great vessels (2 switched vessels) (3) Tricuspid atresia (Tri = 3) (4) Tetralogy of Fallot (Tetra = 4) (5) Total anomalous pulmonary venous return (5 letters in name)

12. Over the past 3 months, a 30-year-old woman has had intermittent…

Adrenal gland

  • Young otherwise healthy woman with episodes of the 5 P’s: elevated blood Pressure, Painful headache, Perspiration, Palpitations and Pallor

13. A 57-year-old woman is brought to the physician 2 days…

Osteoporotic compression fracture

  • Middle-aged woman with risk factor for vertebral compression fracture (long-term prednisone use) presents with sudden onset back pain and tenderness to percussion over the spine, most consistent with a vertebral compression fracture
  • Key idea: Vertebral compression fracture comes in 2 flavors (1) Chronic fracture: Painless with progressive kyphosis and loss of stature (2) Acute fracture: Low back pain with decreased spinal mobility + tenderness at affected level
  • Key idea: Causes of point tenderness over vertebral body include compression vertebral fracture, osteomyelitis and metastatic disease to vertebral bone

14. A previously healthy 32-year-old plumber comes to the…

Muscle strain

  • Young man who developed back pain after lifting heavy equipment and who has paraspinal tenderness (muscles run along either side of spine) with negative straight-leg testing and no neurological symptoms, most consistent with muscle strain (most common cause of back pain in young otherwise healthy people)

15. A 57-year-old man comes to the physician because of a 1-day history…

Indomethacin

  • Middle-aged man who presents with atraumatic right knee arthritis found to have cloudy synovial fluid with a WBC count of 9000 (normal synovial fluid has <200 WBCs) and negatively birefringent, needle-shaped crystals which are classic for acute gout
  • Key idea: Acute gout should be treated with NSAIDs (often indomethacin) and patients should NOT be started on chronic gout drugs (allopurinol, probenecid, etc.) until the acute flare has resolved because these drugs can lead to rapid shifts in uric acid levels that can exacerbate/cause a new flare
  • Note: Patients can sometimes be given oral or intra-articular steroids, but NSAIDs are tried first due to lower risk and good effectiveness in the majority of patients

16. A 67-year-old woman has been intubated for 1 week…

Wean from the ventilator

  • 3 major criteria for extubation of patient includes (1) pH > 7.25 (not retaining CO2) (2) Adequte oxygenation on minimal support (FiO2 40% or less and PEEP 5 mm Hg or less) (3) Intact inspiratory effort and sufficient mental alertness to protect the airway
  • Key idea: As with all other interventions/tubes (Foley, central line, etc.), we only want to use them as long as necessary because they are not harmless interventions
  • Key idea: FiO2 and PEEP are responsible for controlling oxygenation while on ventilator, whereas tidal volume and respiratory rate (which when multiplied equals minute ventilation) are responsible for ventilation (CO2 levels)

17. A 42-year-old man is brought to the physician…

Complex partial seizures

  • Although terminology has fallen out of favor in some neurology spheres, simple seizures lead to no loss of consciousness and complex seizures lead to loss of consciousness (like this patient who has staring spells where he smacks his lips and does not respond)
  • Patient has a partial seizure rather than a generalized seizure because he does not show tonic-clonic movements and his aura of smelling burnt rubber and hearing an intense hissing localizes to the temporal lobe, which is the most common cause/site of partial seizures
  • Important to contrast with absence seizures, which often last for less than 20 seconds (in contrast to 30-90 seconds of partial seizures) and are accompanied by simple automatisms (eyelid fluttering, lip smacking)

18. A 20-year-old African American man with sickle cell…

Increased demand for folic acid

  • Young man with sickle cell anemia who presents with a macrocytic anemia with an abnormally low reticulocyte count, most likely due to folate deficiency
  • Key idea: Folate deficiency often related to (1) Chronic hemolysis (such as in sickle cell disease) (2) Poor dietary intake (such as in alcoholic) (3) Malabsorption (such as in gastric bypass or tropical sprue) and (4) medications (methotrexate, phenytoin, etc.)

19. A 42-year-old man comes to the physician because of a …

Emphysema

  • Young man with family history of “lung and liver problems” and 25 pack-year smoking history presenting with chronic progressive shortness of breath with expiratory wheezes found to have elevated LFTs on exam, most consistent with emphysema in the setting of alpha-1 antitrypsin syndrome
  • Key idea: Alpha-1 antitrypsin is a co-dominant disorder, with homozygous patients developing emphysema and cirrhosis and heterozygous patients being highly prone to emphysema with smoking

20. A healthy 24-year-old woman comes for a routine health…

Repeat examination in 2 weeks

  • Young otherwise healthy woman with no family history of ovarian cancer who presents with a unilateral tender adnexal mass while in the luteal phase of her menstrual cycle, most consistent with a functional ovarian cyst
  • Key idea: Best next step would be repeating the exam during the proliferative phase of the menstrual cycle, where we would expect the mass to go away or improve after ovulation has taken place and the egg has been released
  • Key idea: Patients with a persistent mass or a similar presentation in an older patient should have full work-up performed

21. A previously healthy 87-year-old woman comes to the physician…

Lichen sclerosus

  • Description is most compatible with lichen sclerosus because the vulvar thinning is diffuse (rather than discrete area of pathology seen in patient with vulvar carcinoma)

22. A 37-year-old primigravid woman at 9 weeks’ gestation…

Antiemetic therapy

  • Although it is normal for women to have nausea/vomiting during the first trimester, when they are having severe, persistent vomiting with signs of dehydration and orthostasis, >5% loss of pre-pregnancy weight, or lab findings of ketonuria (pathognomonic of hyperemesis gravidarum > normal pregnancy nausea) then they have hyperemesis gravidarum
  • Patients with hyperemesis gravidarum should be admitted to the hospital and treated with antiemetics and intravenous fluids

23. A 62-year-old woman is brought to the physician by her…

Nitroprusside

  • Patient with blood pressure > 180/20 and papilledema and encephalopathy (confusion, headache, etc.) most consistent with hypertensive encephalopathy
  • Key idea: Potential treatments of hypertensive emergency/encephalopathy include (1) Nitroprusside (2) Labetalol (3) Fenoldopam (4) Clevidipine, nicardipine

24. A previously healthy 47-year-old woman comes to the…

Gallstone pancreatitis

  • Middle-aged woman with risk factors for gallstones (Female, Forty, Fertile) and history consistent with biliary colic who presents with nausea, vomiting, epigastric and RUQ tenderness and lab findings of elevated lipase and amylase most consistent with gallstone pancreatitis
  • Key idea: 2 main causes of pancreatitis are gallstones and alcohol, with elevated triglycerides leading to pancreatitis only when triglyceride levels are >1000-2000
  • Key idea: In setting of gallstone pancreatitis, important to make sure patient does not have concomitant ascending cholangitis (fever, RUQ pain, jaundice, hypotension, altered mental status) because then patient would require urgent ERCP + antibiotics

25. An 18-year-old man comes to the physician 1 week after he…

Operative treatment

  • Young otherwise healthy patient with hypertension should prompt high index of suspicion for a secondary cause of hypertension (fibromuscular dysplasia, hyperaldosteronism, coarctation of the aorta, Cushing’s syndrome, hypo/hyperthyroidism, pheochromocytoma)
  • In this patient who has increased muscular build and stronger pulses in the upper extremities and signs of LV hypertrophy on ECG (secondary to chronically increased afterload), the most likely diagnosis is coarctation of the aorta which requires operative treatment

26. A 6-year-old boy is brought to the physician by his mother

An excess of very long chain fatty acids

  • Child with a genetic X-linked disorder (maternal uncle had similar symptoms) found to have a variety of neural symptoms and diffuse white matter disease on MRI, consistent with Adrenoleukodystrophy (rare peroxisomal disorder of beta-ooxidation that leads to VLCFA buildup in adrenal glands, white matter of brain and testes)

27. A 30-year-old woman comes to the physician for a …

Serum thyroid-stimulating hormone concentration

  • Patient with bipolar disorder who was started on lithium and has experienced weight gain most consistent with a picture of lithium-induced hypothyroidism
  • Key idea: Patients on lithium should have regular TSH monitoring every 6-12 months regardless of symptoms
  • Key idea: Lithium is associated with hypothyroidism, nephrogenic diabetes insipidus, chronic kidney disease and hyperparathyroidism

28. An 80-year-old woman has had bleeding gums for 3 weeks…

C

  • Elderly woman with bland diet with bleeding gums, ecchymoses and perifollicular hemorrhages, which are the classic symptoms seen in scurvy (vitamin C deficiency)
  • Pathophysiology involves problem in collagen synthesis because vitamin C involved in hydroxylation of proline and lysine

29. A 45-year-old woman comes to the emergency department…

Lorazepam

  • Patient’s presentation is consistent with a panic attack (leads to combination of paresthesias, palpitations, abdominal pain, nausea, intensee fear of dying/losing control, light-headedness, chest pain, choking, sweating, shaking and shortness of breath), with panic disorder being acutely treated with benzo’s (such as lorazepam) and prophylactically/chronically treated with SSRIs
  • Note: Patients often develop tingling around the lips secondary to hypocalcemia which occurs because patients begin to breathe rapidly and deeply, leading to a respiratory alkalosis, which leads to more calcium binding to negatively-charged albumin, leading to a drop in free calcium

30. A healthy 23-year-old woman, gravida 1, para 1…

Oral levonorgestrel now and again in 12 hours

  • Four options for emergency contraception include (1) Copper IUD (most effective) (2) Ulipristal (3) Levonorgestrel (“Plan B”) (4) Oral contraceptives

31. A 5-year-old girl is brought to the physician because…

Empyema

  • Child who has been incompletely treated for pneumococcal pneumonia who presents with high fever, nonproductive cough and diminished breath sounds over lower lung fields with dullness to percussion (suggestive of some form of pleural effusion), most consistent with an empyema
  • Bronchopleural fistula often seen after trauma, lung abscess often seen secondary to aspiration, pleurodynia leads to sudden sharp chest pain and pneumothorax would lead to diminished breath sounds with hyperresonance to percussion

32. A 67-year-old woman is brought to the emergency department…

Esophageal perforation

  • Older woman who received an upper endoscopy with associated procedure presents with severe chest pain, hematemesis, crepitus in the neck and positive blood in the GI tract most consistent with an esophageal perforation
  • Key idea: Best diagnostic test would be esophagography with water-soluble contrast and patient should be treated with IV antibiotics, PPIs and emergency surgery
  • Contrast with Mallory-Weiss tear, which is a partial tear and therefore would not lead to crepitus and would often lead to more profound hematemesis

34. A 22-year old primigravid woman at 16 weeks’ gestation is…

Choriocarcinoma

  • Young pregnant woman who presents with enlarged uterus (uterus to umbilicus consistent with gestational age of 20+ weeks), absent fetal heart tones, severely elevated beta0hCG and chest x-ray showing multiple densities (“cannonball metastases”) most consistent with choriocarcinoma
  • Key idea: Rare diagnosis that can develop during pregnancy, after delivery or after abortion
  • Key idea: Can be associated with symptoms related to increased hCG due to alpha subunit of hCG being the same as the alpha subunit of LH, FSH and TSH, including early pre-eclampsia, hyperemesis gravidarum, hyperthyroidism and theca-lutein cysts

35. An asymptomatic 21-year-old woman is found to have an…

Benign cystic teratoma

  • Image demonstrates serosal lining, hair and a tooth/bone and therefore is consistent with a teratoma (tumor made up of several different types of tissue)
  • Key idea: Most common cause of an adnexal mass in a young woman is a teratoma, and it can be associated with ovarian torsion

36. A 13-year-old boy is brought to the physician by his mother…

No pharmacotherapy is indicated

  • Children with hypertension (BP > 95th percentile) are primarily treated with lifestyle changes (weight loss, low-sodium diet, regular physical activity)
  • Indications for treatment of hypertension in children: (1) Symptomatic hypertension (headaches, etc.) (2) secondary hypertension (3) end-organ damage (4) diabetes (5) hypertension refractory to lifestyle changes
  • Key idea: In both children and adults, lifestyle interventions should be thoroughly tried before getting medications on board in setting of hypertension

37. A 72-year-old man comes to the physician because of a …

Oral enalapril therapy

  • Elderly man with chronic diabetes found to have signs of CKD (elevated BUN and creatinine) and albuminuria who could have benefitted from an ACE inhibitor or ARB
  • Key idea: Two most common causes of CKD include hypertension and diabetes, with diabetes leading to nephropathy because it leads to preferential arteriosclerosis of the efferent arteriole > afferent arteriole, thus leading to high pressure in glomerulus, resulting elevated GFR and damage to glomerulus over time (ACE inhibitor or ARB can reduce this damage by dilating the afferent arteriole, leading to a decreased GFR)
  • Key idea: ACE inhibitors or ARBs should be initiated at onset of microalbuminuria in all diabetic patients

38. A 57-year-old man is brought to the physician by his wife…

Withold the results as the patient wishes

  • Patient’s have the right to decline when offered information about their health or lab results

39. A previously healthy 3-month-old girl is brought to…

Dysmorphogenesis of the third and fourth pharyngeal pouches

  • Infant presenting with grunting and breathing difficulty found to be hypocalcemic with absent thymic shadow and infection with an organism commonly seen in patients with HIV and other forms of cellular (T cell) deficiency, most consistent with DiGeorge syndrome
  • Mnemonic for symptoms for CATCH-22: Cardiac abnormalities (often Tetralogy of Fallot), Abnormal facies, Thymic aplasia, Cleft palate/lip, Hypocalcemia (secondary to parathyroid a/dysgenesis)

40. A 67-year-old man is brought to the emergency department…

Brain abscess

  • Elderly patient with a history of aortic valve replacement presents with fever, new systolic murmur and signs of septic emboli to the brain, most consistent with infective endocarditis
  • Although brain abscess is a rare complication of endocarditis (1-7%), it is a feared complication of septic emboli to the brain and is more likely to occur than the other options listed

41. A 32-year-old man comes to the physician because of…

Vitamin D deficiency

  • Young patient with malabsorption (intermittent diarrhea with weight loss and fat in the stool) who has low calcium and phosphorous levels, which is most consistent with vitamin D deficiency
  • Key idea: Vitamin D responsible for calcium and phosphorous absorption from the GI tract (contrast with PTH which leads to increased calcium and decreased phosphate)
  • Key idea: Fat soluble vitamins are vitamins A, D, E and K

42. A 4-week-old newborn is brought to the physician because…

High pulmonary vascular resistance

  • Newborn presents with labored breathing with cardiomegaly, increased pulmonary vascular markings, RVH on ECG and a holosystolic murmur at the lower left sternal border that was not present at birth, which is most consistent with a ventricular septal defect
  • Key idea: In utero the pulmonary vascular resistance is very high because the lungs are not participating in gas exchange and as the baby acclimates to the outside world the pulmonary vascular resistance steadily drops; because of this, left-to-right murmurs can increase in severity over the first several weeks of life because blood meets less resistance moving from the LV to the RV to the pumonary arteries
  • Key idea: Blood (like most things in this world) will follow the path of least resistance!

43. A 32-year-old woman comes to the physician because of bright…

Anesthetic ointment and stool softeners

  • Young healthy woman who has bloody stools with severe pain and an anal fissure on physical exam, most consistent with an anal fissure
  • Key idea: Most anal fissures can be managed conservatively with anesthetic ointment and stool softeners (highly associated with mild constipation, as is the case with our patient)

44. A 67-year-old woman comes for a routine health maintenance…

Measurement of serum parathyroid hormone conentration

  • Healthy patient with increased calcium and decreased phosphorous, most consistent with elevated PTH levels with the most common cause being primary hyperparathyroidism
  • Key idea: Parathyroid hormone leads to increased calcium and decreased phosphorous, whereas vitamin D leads to increased calcium and increased phosphorous
  • Key idea: If patient had a positive protein gap (total protein – albumin > 4 g/dL), then you would have higher index of suspicion for multiple myeloma and would perform a serum protein electrophoresis

45. One month after undergoing an uneventful renal…

Increased dosage of corticosteroids

  • Patient with renal transplant who develops increased BUN and creatinine on order of weeks to months most concerning for acute transplant rejection (which is confirmed by biopsy in this patient)
  • Key idea: First-line treatment for acute rejection are steroid boluses vs. antilymphocyte agents vs. antithymocyte serum

46. A 72-year-old man comes to the physician with his wife…

Measurement of blood lead concentration

  • Patient with risk factor for lead poisoning (homemade whiskey) presents with abdominal pain, neurologic symptoms, and a microcytic anemia, which are the three classic symptoms of lead poisoning
  • Key idea: Potential buzzwords that should trigger thought of lead poisoning include battery factory, homemade whiskey, renovating old house, etc.
  • Note: While not a high-yield association, lead poisoning can lead to worsening of gout

We are not affiliated with the NBME, USMLE or AAMC.

The answer explanations may not be reproduced or distributed, in whole or in part, without written permission of Step Prep.